PT 43 Sec 3 Problem 25... Wealth is... Forum

Prepare for the LSAT or discuss it with others in this forum.
Post Reply
nmare

New
Posts: 41
Joined: Sat Jul 23, 2011 1:44 am

PT 43 Sec 3 Problem 25... Wealth is...

Post by nmare » Fri Sep 02, 2011 5:22 pm

Correct Answer: A
My Answer: D

I can't explain how A is correct, although know the reason D is incorrect. Can anyone help out and explain how A is correct?

Thanks

barneytrouble

Bronze
Posts: 239
Joined: Wed Aug 17, 2011 11:43 pm

Re: PT 43 Sec 3 Problem 25... Wealth is...

Post by barneytrouble » Sat Sep 03, 2011 3:02 am

nmare wrote:Correct Answer: A
My Answer: D

I can't explain how A is correct, although know the reason D is incorrect. Can anyone help out and explain how A is correct?

Thanks
OK set up a venn diagram for each argument.

The prompt's would go:
Circle 1: Good Things Circle 2: Things that cause harm (These circles do not overlap at all)
You then have "wealth is often harmful" so take the object "wealth" and put it in Circle 2 - the tricky part here is that even though where the premise says "no harm at all" which means that if ANY part of the "wealth" object was inside Circle 2, it can not be in Circle 1.
Therefore, wealth can not be a part of circle 1

For answer choice A:
Circle 1: People in chess club Circle 2: People who love to golf (Again, these circles do not overlap at all)
You then have "Alex loves to golf" so take object "Alex" and put him in Circle 2
You now end up with "anything in circle 2 can not be a part of circle 1"

To summarize:
If wealth is often harmful to people, it can NOT in any case have overlap with good things, since good things cause no harm whatsoever (even 1% !!!). My guess is you got tripped up by thinking the sentence "wealth is often harmful to people" left the possibility that your "Wealth" circle could be partly in "Things that cause harm" and partly in "Good Things"
Basically, anything that has even 1% of their circle in the "Things that cause harm" circle can in no way whatsoever have any overlap with the "Good Things" circle. The final symbolization would be: Circle 1 and Circle 2 have no things in common, X is a part of Circle 2, therefore X is not a part of Circle 1.

User avatar
EarlCat

Silver
Posts: 606
Joined: Mon Mar 12, 2007 4:04 pm

Re: PT 43 Sec 3 Problem 25... Wealth is...

Post by EarlCat » Sat Sep 03, 2011 1:10 pm

nmare wrote:Correct Answer: A
My Answer: D

I can't explain how A is correct, although know the reason D is incorrect. Can anyone help out and explain how A is correct?

Thanks
Stimulus
Premise: H --> ~G
Premise: W --> H
Conclusion W --> ~G

Answer A
Premise: G --> ~C
Premise: A --> G
Conclusion: A --> ~C

Post Reply

Return to “LSAT Prep and Discussion Forum”